Calculating the Inner Product of <2011|0011>

In summary, The inner product <2011|0011> is equal to zero because the labels do not match up and the result is trivially zero. In order to solve this question, you would need to use the Clebsch-Gordan equation and look up a table to write the state in terms of the |l_1,l_2,m_{l,1} m_{l,2}> states. However, since the labels are ordered correctly and l_total = 0 and l_1 = 2 and l_2=0, the result is trivially zero.
  • #1
UrbanXrisis
1,196
1

Homework Statement



What s the inner product [tex]<2011|0011>[/tex]

Homework Equations



[tex]C_{m_1m_2}=<l_1l_2m_1m_2|lml_1l_2>[/tex]

The Attempt at a Solution



I'm not sure how to exactly solve this question. The first thing that came to my mind was the Clebsch-Gordan equation, since that's what it looks like, but then I saw that it doesn't really make any sense because of this:

[tex]|lml_1l_2>=|0011>[/tex]

[tex]|l_1l_2m_1m_2>=|2011>[/tex]

I'm guessing the inner product is zero, but I'm not sure how to show this.

maybe because since m=0, m_1=1 and m_2=1 wouldn't make sense? not too sure, any help would be appreciated.

thanks,
ux
 
Physics news on Phys.org
  • #2
UrbanXrisis said:

Homework Statement



What s the inner product [tex]<2011|0011>[/tex]

Homework Equations



[tex]C_{m_1m_2}=<l_1l_2m_1m_2|lml_1l_2>[/tex]

The Attempt at a Solution



I'm not sure how to exactly solve this question. The first thing that came to my mind was the Clebsch-Gordan equation, since that's what it looks like, but then I saw that it doesn't really make any sense because of this:

[tex]|lml_1l_2>=|0011>[/tex]

[tex]|l_1l_2m_1m_2>=|2011>[/tex]

I'm guessing the inner product is zero, but I'm not sure how to show this.

maybe because since m=0, m_1=1 and m_2=1 wouldn't make sense? not too sure, any help would be appreciated.

thanks,
ux

Are you sure that your labels are ordered correctly?

Yes, it's a CG question, but here the result is trivially zero. In pricniple, you would have to look up a table and write the state [itex] | l_{total} =0, m_{l,total} =0, l_1 =1, l_2=1> [/itex] in terms of the [itex] |l_1,l_2,m_{l,1} m_{l,2}>[/itex] states.


But if your labels are ordered correctly and l_total = 0 and l_1 = 2 and l_2=0 then the result is trivially zero since adding 2 and 0 only gives a total l equal to 2 (and the m quantum numbers don't match either. if m_1=1 and m_2 =1, then m_total must be 2).

Patrick
 

Related to Calculating the Inner Product of <2011|0011>

1. What is the inner product?

The inner product is a mathematical operation that takes two vectors and produces a scalar value. It is also known as the dot product or scalar product.

2. How do you calculate the inner product?

The inner product is calculated by multiplying the corresponding components of the two vectors and then summing up the products. In other words, it is the sum of the products of the first components, second components, and so on.

3. What is the significance of the inner product?

The inner product has various applications in mathematics, physics, and engineering. It can be used to find the angle between two vectors, determine if two vectors are orthogonal, and project one vector onto another.

4. Can the inner product be negative?

Yes, the inner product can be negative. It is negative when the angle between the two vectors is greater than 90 degrees.

5. How is the inner product related to vector length?

The inner product is related to vector length through the concept of orthogonality. If two vectors are orthogonal, their inner product is zero, and the vectors are perpendicular to each other. Therefore, the inner product can be used to determine the length of a vector in a particular direction.

Similar threads

Replies
16
Views
2K
  • Quantum Physics
Replies
8
Views
2K
  • Advanced Physics Homework Help
Replies
4
Views
2K
  • Advanced Physics Homework Help
Replies
3
Views
2K
Replies
17
Views
2K
  • Advanced Physics Homework Help
Replies
3
Views
1K
  • Advanced Physics Homework Help
Replies
20
Views
4K
  • Calculus and Beyond Homework Help
Replies
5
Views
3K
  • Advanced Physics Homework Help
Replies
4
Views
4K
Replies
5
Views
1K
Back
Top